2
$\begingroup$

Let $\Omega$ be locally compact topological space. Consider $C_0(\Omega)$ being the space of continuous functions $u$ on $\Omega$ that vanish at infinity, that is, for $r>0$ there is a compacte set $K\subset\Omega $ such that $\sup_{\Omega\setminus \Omega}|u(x)|<r$. The dual of $C_0(\Omega)$ is well-known to be $M_b(\Omega)$ the space of signed bounded Radon measures $\Omega$.

For $u\in C_0(\Omega)$ with $\|u\|_\infty\neq 0$, consider the duality mapping (which is a convex set)

$$J(u) =\{ \mu\in M_b(\Omega)\,\,:\,\, \langle\mu, u\rangle= \|u\|_\infty,\,\,\|\mu\|=1\}$$

I would like to show that if $\mu\in J(u)$ is an extreme point then there exists $a\in \Omega$ such that $$|u(a)|=\|u\|_\infty\qquad \text{and}\qquad \mu= \text{sign}_0(u(a))\delta_a$$ where $\delta_a$ is the Dirac measure at $a\in \Omega$.

I was able to show that $ \mu= \text{sign}_0(u(a))\delta_a$ are extreme points of $J(u)$ using this answer here: \url{https://math.stackexchange.com/q/2303204}

Recall that $\mu$ is an extreme point if and only if $J(u)\setminus\{\mu\}$ is also a convex set.

Any reference or answer is welcome.

$\endgroup$
0

1 Answer 1

2
$\begingroup$

$\newcommand\Om\Omega\newcommand\de\delta$We have to show the following:

Suppose that $\Om$ is a locally compact topological space, $0\ne u\in C_0(\Om)$, and $\mu$ a signed bounded Radon measure over $\Om$ that is an extreme point of $J(u)$. Then $\mu=\pm\de_z$ for some choice of the sign and some $z\in\Om$.

Without loss of generality (wlog), by rescaling, $\max_\Om|u|=\|u\|_\infty=1$. Since $\mu$ is Radon, the topological space $\Om$ must be Hausdorff. Let $(\Om_+,\Om_-)$ be a Hahn decomposition corresponding to the signed measure $\mu$ and let $(\mu_+,\mu_-)$ be the Jordan decomposition of $\mu$. Let $u_\pm:=\max(0,\pm u)$. Then $$1=\int_\Om u\,d\mu=\int_\Om u_+\,d\mu_+ +\int_\Om u_-\,d\mu_- -\int_\Om u_+\,d\mu_- -\int_\Om u_-\,d\mu_+ \\ \le \int_\Om u_+\,d\mu_+ +\int_\Om u_-\,d\mu_-\le1.$$ Therefore and because $1=\|\mu\|=|\mu|(\Om)=\mu_+(\Om) +\mu_-(\Om)$, we have $$u_\pm=\pm1\quad\text{$|\mu|$-almost everywhere on }\Om_\pm.$$

Note: Any nonzero Radon measure $\nu$ over $\Om$ has at least one support point. Indeed, otherwise, for some compact $K\subseteq\Om$ such that $\nu(K)>0$ and for each $z\in K$ there is an open neighborhood $U_z$ of $z$ with $\nu(U_z)=0$. Covering now $K$ by finitely many $U_z$'s, we get $\nu(K)=0$, a contradiction.

So, the support $S_\mu$ of $|\mu|$ is nonempty. Suppose there are two distinct points $x$ and $y$ in $S_\mu$. Let $U$ and $V$ be two disjoint open neighborhoods of $x$ and $y$, respectively. Then $M:=|\mu|(U)>0$, $N:=|\mu|(V)>0$, and $$\int_U u\,d\mu+\int_V u\,d\mu =\int_U |u|\,d|\mu|+\int_V |u|\,d|\mu| =|\mu|(U)+|\mu|(V)=M+N.$$ Take any any $t\in(0,\min(M,N))$ and let $$\mu^\pm(A):=\frac{M\pm t}M\,\mu(A\cap U) +\frac{N\mp t}N\,\mu(A\cap V)+\mu(A\setminus U\setminus V)$$ for Borel $A\subseteq\Om$. Then $\mu^\pm\in J(u)$, $\mu^+\ne\mu^-$, and $\mu=\frac12\,(\mu^+ +\mu^-)$, which contradicts the assumption that $\mu$ is an extreme point of $J(u)$.

So, $S_\mu=\{z\}$ for some $z\in\Om$. If the "restriction" of $|\mu|$ to $\Om\setminus\{z\}$ were nonzero, then, by the Note, $|\mu|$ would have a support point in $\Om\setminus\{z\}$. So, the "restriction" of $|\mu|$ to $\Om\setminus\{z\}$ is zero. Recalling that $\|\mu\|=1$, we get the desired conclusion $\mu=\pm\de_z$. $\quad\Box$.

$\endgroup$

You must log in to answer this question.

Start asking to get answers

Find the answer to your question by asking.

Ask question

Explore related questions

See similar questions with these tags.